Module 1 Qualified Plans Requirements and Regulatory Plan Considerations Flashcards

1
Q

Susan makes 400k $ working for Great Grapes Inc. She defers 4% into the 401(k) and receives the 4% match. How much will go into her account in 2023?

A)
$56,400
B)
$32,000
C)
$26,400
D)
$22,500

A

C $26,400

Only the first $330,000 of compensation may be used to determine contributions to qualified retirement plans in 2023. Thus, she contributes 4% of $330,000 in 2023. This amount is MATCHED, so that ups the total percentage contributed to 8% (of $330k) and she gets $26,400 in her 401(k) that year.

How well did you know this?
1
Not at all
2
3
4
5
Perfectly
2
Q

Qualified retirement plans have which of the following characteristics?

They are subject to ERISA requirements.
They offer tax deferred earnings to employees
They can discriminate in favor of highly compensated employees
They provide tax deduction for the employer only when the employee is income taxed on the money

I and II
B)
III and IV
C)
I, II, III, and IV
D)
I, II, and III

A

Statements I and II are correct.
Qualified retirement plans are subject to ERISA requirements.
Qualified retirement plans offer tax-deferred earnings to employees. Example of this 401(k), you’re not taxed on your contributions (or earnings) until you take it out. Then you’re taxed on it at an income level I think.

Qualified retirement plans are subject to ERISA requirements and provide tax deferral on investment earnings for employees. While qualified plans in general can provide different levels of benefits to different classes of employees, qualified plans cannot “discriminate in favor of highly compensated employees” in the sense that there is a legal limit to the amount of the difference. As long as the difference is inside the legal limits, the plan is not discriminatory (by definition). Qualified retirement plans provide an immediate tax deduction on employer contributions.

How well did you know this?
1
Not at all
2
3
4
5
Perfectly
3
Q

Which of the following are included in the annual additions limit for defined contribution plans?

Investment gain
Employee elective deferral contributions
Employer contributions
Forfeitures reallocated to the remaining participants in the plan

A)
II, III, and IV
B)
II and III
C)
I and IV
D)
I, II, III, and IV

A

II III IV.

How well did you know this?
1
Not at all
2
3
4
5
Perfectly
4
Q

Which statement regarding qualified retirement plans is CORRECT? Qualified plans…

A)
are subject to ERISA requirements.
B)
offer taxable earnings to employees each year.
C)
provide a deferred tax deduction for employer funding.
D)
can discriminate in favor of highly compensated employees.

A

Qualified plans are subject to ERISA requirements.

They provide tax deferral on investment earnings for employees. While qualified plans in general can provide different levels of benefits to different classes of employees, qualified plans cannot discriminate in favor of highly compensated employees.

How well did you know this?
1
Not at all
2
3
4
5
Perfectly
5
Q

ERISA requirements for qualified plans include:

A)
all of these.
B)
reporting and disclosure.
C)
participation and fiduciary requirements.
D)
coverage and vesting.

A

All of these

How well did you know this?
1
Not at all
2
3
4
5
Perfectly
6
Q

Which of these individuals are “key employees” as defined by the internal revenue code?

A more than 5% owner of the employer company
An employee who received compensation of more than $150k from the employer (for 2023)
An officer of the employer who received compensation of more than $215,000 in 2023
A 1% owner of the employer with an annual compensation from the employer of more than $147,000

A)
I and III
B)
II, III, and IV
C)
II and IV
D)
I and II

A

I and III
A more than 5% owner of the company
An officer of employer who earned more than $215k (2023)

Option II is one definition of a “highly compensated employee”.

Both II and IV are incorrect because they do not fall within one of the following definitions of a key employee:
-An officer whose annual compensation from the employer exceeds 215k (2023)
-A more than 5% owner of the employer company
-A more than 1% owner of the employer company having annual compensation from the employer in excess of $150k

How well did you know this?
1
Not at all
2
3
4
5
Perfectly
7
Q

If a defined contribution plan is top heavy, the required minimum contribution is usually equal to what percentage of each non key participant’s compensation?

A

If a defined contribution plan is top heavy, the min contribution to the plan is 3% per year unless thee key employees are receiving less than 3%. If the key employees are getting less than 3% contributions, then the non key employees must at least get what the key employees are getting

How well did you know this?
1
Not at all
2
3
4
5
Perfectly
8
Q

Which of the following does NOT meet the definition of active participation in a retirement plan for the purposes of determining the deductibility of IRA contributions?

A)
Kerry has benefits accrued for him under his employer’s defined benefit pension plan.
B)
Matt makes contributions to his employer’s qualified plan.
C)
An employee is eligible to defer salary to his employer’s Section 401(k) plan.
D)
Betty has employee forfeitures reallocated to her account by her employer but makes no elective deferrals in the same plan year.

A

C.

If you got it wrong - note that the question asks about IRA contribution, and C tells you about 401(k) contributions. Paragraph below says this: you don’t have to contribute to be considered active for 401(k), BUT for an IRA you have to contribute to be considered active.

For the Section 401(k) retirement plan, an employee is considered covered as long as he or she is eligible to defer part of his compensation; he or she does not have to actually make contributions. Note, however, that active participation for purposes of determining deductibility of IRA contributions differs from being covered under a qualified plan for nondiscrimination testing. Specifically, an employee must be contributing to the plan, having employer contributions to the plan, forfeitures reallocated on his or her behalf, or accruing a defined benefit before he or she is considered an active participant in a qualified plan for IRA purposes. Plan earnings alone do not make someone an active participant. Also, receiving retirement plan benefits do not make someone an active participant.

How well did you know this?
1
Not at all
2
3
4
5
Perfectly
9
Q

Which of the following types of retirement plans allow integration with Social Security?

Defined benefit pension plan
Simplified employee pension (SEP) plan
Savings incentive match plan for employees (SIMPLE)
Employee stock ownership plan (ESOP)
A)
II and III
B)
I, II, III, and IV
C)
I only
D)
I and II

A

I and II.

Defined benefit pension plans and Simplified employee pension plans (SEP) can be integrated with social security.

ESOPS and SIMPLE plans cannot be integrated with social security.

How well did you know this?
1
Not at all
2
3
4
5
Perfectly
10
Q

If a qualified contribution retirement plan is found to meet the requirements under the Employee Retirement Income Security Act of 1974 (ERISA) and the IRS regulations, which of these statements is correct?

A)
Employees are not taxed on plan contributions or earnings attributed to plan contributions as long as a plan distribution does not occur.
B)
The employer is permitted to require three years of service eligibility as long as employer contributions are immediately 100% vested to participants.
C)
The participating employees must petition the IRS for acceptance of the plan.
D)
The PBGC will provide coverage to the plan.

A

A. Employees are not taxed on plan contributions or earnings attributed to plan contributions as long as a plan distribution does not occur.

A “two years of service” eligibility requirement is permitted if 100% immediate vesting is also used. Employees do not petition the IRS for acceptance of the plan. No defined contribution plans are eligible for PBGC coverage.

How well did you know this?
1
Not at all
2
3
4
5
Perfectly
11
Q

Which of the following are minimum coverage tests for qualified retirement plans?

Minimum allowed discrimination
Average benefits percentage test
Ratio test
Maximum compensation test

A)
II, III, and IV
B)
I, II, and III
C)
II and III
D)
I and II

A

II and III

The two minimum coverage tests for qualified retirement plans are the average benefits percentage test and the ratio test. To be qualified, a retirement plan must meet at least tests if the plan does not meet the percentage (safe harbor) test.

How well did you know this?
1
Not at all
2
3
4
5
Perfectly
12
Q

Claire is the HR director for major marketing inc. she wants to implement a new qualified retirement plan for the company. there are numerous fed guidelines with which the company must comply. Which of the following fed agencies is tasked with supervising the creation of new qualified retirement plans?

IRS
DOL
PBGC
ERISA

A

IRS

How well did you know this?
1
Not at all
2
3
4
5
Perfectly
13
Q

Which are agencies that administer and ensure compliance with fed laws that apply to qualified retirement plans?

Department of Labor (DOL)
Employee Retirement Income Security Agency (ERISA)
IRS
Pension Benefit Guaranty Corporation (PBGC)

A)
II and IV
B)
I, II, and III
C)
I and III
D)
I, III, and IV

A

I III IV

Under the reporting and disclosure requirements of the “Employee Retirement Income Security ACT of 1974 (ERISA)”, annual reports and summary plan descriptions are filed with the IRS (which provides copies to the DOL). An annual premium payment form is filed with the PBGC. If the defined benefit plan is not in compliance with funding requirements, the PBGC can terminate the plan.

How well did you know this?
1
Not at all
2
3
4
5
Perfectly
14
Q

Dorban products inc, has an annual payroll of $800k. John, the president, wishes to make the maximum contribution to the integrated profit sharing plan this year. What is the amount?

A)
$80,000
B)
25% of base compensation plus 30.7% of excess compensation totaling $245,600
C)
$200,000
D)
$120,000

A

C

The max contribution is 25% of covered payroll, or $200,000

How well did you know this?
1
Not at all
2
3
4
5
Perfectly
15
Q

ERISA requires reporting and disclosure of plan information to all of the following except:

DOL
SEC
IRS
Plan participants

A

SEC. Why would they care?

How well did you know this?
1
Not at all
2
3
4
5
Perfectly
16
Q

RQZ company employs 200 nonexcludible employees, 20 of whom are highly compensated. 16 of these 20 and 125 of the non-highly compensated employees benefit from the RQZ qualified pension plan. The average benefits accrued fro the highly compensated is 8%. The ratio test for the plan just listed is:
A)
86.8%.
B)
80.0%.
C)
69.4%.
D)
70.0%.

A

A.

20 of the 200 are highly compensated and 16 of those 20 are on the pension plan. 16/20 = 80%

180 of the 200 are non highly compensated. And 125 of those are on the pension plan. 125/180 = 69.44%

69.44% / 80& = 86.8%
The passing percentage for the ratio coverage test is 70%, so it passes.

How well did you know this?
1
Not at all
2
3
4
5
Perfectly
17
Q

Which is Correct in describing the integration of a 5% money purchase plan?

A)
The maximum excess contribution percentage is 10.0%.
B)
The maximum excess contribution percentage is 15.7%.
C)
The permitted disparity is 5.7%.
D)
The offset integration method may be used.

A

A
The max excess contribution percentage is 10%.

A MPP with a base contribution percentage of 5% will have a 5% permitted disparity (lesser of the base (5%) or 5.7%). Therefore the max excess contribution percentage will be 10% (5% base + 5% permitted disparity).

How well did you know this?
1
Not at all
2
3
4
5
Perfectly
18
Q

What are the requirements and effect of an eligible investment advice arrangement under the Pension Protection Act?

An eligible investment advice arrangement allows a plan fiduciary to give advice including recommending their own proprietary funds without violating fiduciary rules.
The investment adviser’s fees must be neutral.
An unbiased computer model certified by an independent expert to create a recommended portfolio for the client’s consideration is used.
Investment advisors for IRAs may only use the unbiased computer model option when providing eligible investment advice.
A)
I, III, and IV
B)
I, II, and III
C)
I and II
D)
II and III

A

I II III

IV is wrong. Investment advisors for IRAs may only use the neutral fee option and not the computer model option when providing eligible investment advice.

How well did you know this?
1
Not at all
2
3
4
5
Perfectly
19
Q

Which regarding top heavy plan is correct?

A top heavy defined benefit pension plan must provide accelerated vesting.
A top heavy plan is one that provides more than 50% of its aggregate accrued benefits or account balances to key employees.
A top heavy defined pension benefit plan must provide a min benefit accrual of 2% multiplied by the number of years of service (up to 20%).
For a top-heavy defined contribution plan, the employer must make a minimal contribution of 3% of annual covered compensation for each eligible non-key employee. If the contribution percentage for key employees is less than 3%, the contribution percentage to non-key employees can be equal to the key employees’ percentage.

A

I III IV

II is wrong, instead of more than 50% it’s MORE THAN 60%. 60% exactly does NOT make a plan top heavy.

How well did you know this?
1
Not at all
2
3
4
5
Perfectly
20
Q

Qualified retirement plans should do which of the following?

They must meet specific vesting requirements
They have special tax advantages over nonqualified plans
They must provide definitely determinable benefits
They require an annual profit to allow funding for the plan

A

I II III

IV is incorrect, annual profit is not required for a qualified plan to be funded.

How well did you know this?
1
Not at all
2
3
4
5
Perfectly
21
Q

Which of the following is correct about the permitted (SS integration) rules for DB plans?

A plan that provides a benefit for wages up to the integration level, plus a higher benefit for wages that exceed the integration level, is an integrated defined benefit excess plan.
A plan that provides that an employee’s benefit otherwise computed under the plan formula is reduced by a fixed amount or formula amount in relationship with the person’s Social Security benefit is an integrated defined benefit offset plan.
Covered compensation is the average of the participant’s compensation not in excess of the taxable wage base for the three-consecutive-year period ending with or within the plan year.
The base benefit percentage is determined by calculating the benefits provided by the plan based on compensation below the integration level, and expressing these benefits as a percentage of compensation below the integration level.

A

I II IV.

III is wrong because covered compensation means the average social security taxable wage base over the last 35 years.

How well did you know this?
1
Not at all
2
3
4
5
Perfectly
22
Q

Which of the following fed agencies is tasked with supervising the creation of new qualified retirement plans?

SEC
IRS
DOL
PBGC

A

IRS

How well did you know this?
1
Not at all
2
3
4
5
Perfectly
23
Q

Which of the following statements regarding prohibited transactions by a fiduciary or an individual associated with traditional IRA accounts are CORRECT?

Generally, if an individual or the individual’s beneficiary engages in a prohibited transaction with the individual’s IRA account at any time during the year, it will not be treated as an IRA as of the first day of the year.
If an individual borrows money against an IRA annuity contract, the individual must include in gross income the fair market value of the annuity contract as of the first day of the tax year.
Selling property to an IRA by a fiduciary or an individual owner of the IRA is not prohibited.
A 50% penalty will be assessed against an IRA owner who borrows money against her IRA.
A)
II and III
B)
I, III, and IV
C)
I, II, and III
D)
I and II

A

Statements I and II correctly describe prohibited transactions. Statement III is incorrect. Selling property to an IRA by a fiduciary or an individual owner of the IRA is a prohibited transaction. Statement IV is incorrect. The individual may have to pay the 10% additional tax on premature distributions. Also, if a person uses an Individual Retirement Account (not an Individual Retirement Annuity) as collateral for a loan, then only the amount of the Individual Retirement Account collateralized will be considered distributed. While this is bad, it is not as bad as the treatment of of a collateralized Individual Retirement Annuity in which the entire collateralized account is deemed to be distributed.

How well did you know this?
1
Not at all
2
3
4
5
Perfectly
24
Q

For 2023, the maximum annual contribution under a money purchase pension plan on behalf of a participant is the lesser of 100% of the employee’s covered compensation, or

A)
$265,000.
B)
$66,000.
C)
$330,000.
D)
$22,500.

A

66k

How well did you know this?
1
Not at all
2
3
4
5
Perfectly
25
Q

Brad Elberly has been the sole owner and operator of Woodmasters Inc. for the past 15 years. Brad is age 45, and his salary from the business is $130,000. Brad and his wife, Laura, want to retire when Brad is age 65. Relevant information regarding the business is summarized below:

Financial performance fluctuated over the first 10 years.
Cash flow and profits have stabilized during the past five years and are expected to show modest but consistent growth in the future. Excess cash flow of approximately $150,000 is expected to be available this year. Future years should be about the same. Brad has expressed some concern about the company’s outdated equipment and is considering renovating the plant and replacing the outdated equipment over the next five years. The total cost should be about $300,000.
Total compensation for all employees (including Brad) is $245,000.
The four full-time rank-and-file employees range from age 19 to age 38, and have been with Woodmasters for periods ranging from four months to six years. Age and service information is shown below:
Employee Age Completed Years of Service Compensation
Brad 45 15 years $130,000
Beth 38 6 years $40,000
Todd 27 6 months $25,000
Carol 30 2 years $28,000
Jim 19 4 months $22,000
Assume that Brad installs a 10% money purchase plan at Woodmasters this year. The plan provides for a graded vesting schedule. Which of the following statements will be true regarding this money purchase plan?

The plan will be top-heavy.
The plan will not be top-heavy.
An additional 3% must be contributed for the non-key employees.
The plan will be discriminatory due to the ratio of Brad’s salary to the other employees’ salaries.

A

A I only

In the first year of a qualified plan that uses a compensation-based allocation formula (as a money purchase plan does), it is possible to determine whether the plan will be top-heavy based on the key employees’ compensation as a percentage of covered payroll. Brad’s salary of $130,000 is 65% of the $198,000 covered payroll; since this exceeds 60%, the plan is top-heavy. However, since the plan already contributes 10% no additional contribution is required and vesting need not be accelerated since defined contribution plans must use top heavy vesting.

How well did you know this?
1
Not at all
2
3
4
5
Perfectly
26
Q

Which of these statements regarding a top-heavy retirement plan is false?

A)
Top-heavy plans favor key employees by providing more than 60% of the plan benefits to these employees.
B)
Defined benefit pension plans can be top heavy; defined contribution plans cannot.
C)
Close scrutiny of the top-heavy rules is desirable from a planning standpoint because an understanding of these rules coupled with effective personnel decisions and plan design may enable the plan to escape top-heavy status.
D)
Small-business owners are prone to shaping the organization’s retirement plan primarily to shelter taxes for themselves and key employees, thereby creating top-heavy plans.

A

B

Both DB and DC plans can be top heavy, but both must satisfy rules that apply to top heavy plans

Also worth noting: this is what top heavy means: it favors key employees by providing more than 60% of the plan benefits to them.

How well did you know this?
1
Not at all
2
3
4
5
Perfectly
27
Q

Who of the following is a fiduciary for the XYZ Qualified Pension Plan?

Joe, the administrator for XYZ’s pension plan
Bill, the investment manager for XYZ’s pension plan
Mary, a CFP® professional, and the paid investment adviser of the XYZ Qualified Pension Plan
Ralph, the XYZ owner who selected Joe, Bill, and Mary
A)
I, II, and IV
B)
I, II, III, and IV
C)
I and III
D)
II and III

A

All

The administrator for the pension plan
The investment manager for the pension plan
The CFP(R) professional for the pension plan
The owner of the company who select all of the above for the plan

How well did you know this?
1
Not at all
2
3
4
5
Perfectly
28
Q

An integrated defined benefit plan providing a 20% flat benefit could provide

A)
a permitted disparity no greater than 25.7%.
B)
a permitted disparity of 26.5% and excess benefit of 40%.
C)
an excess benefit of 26% and 40% permitted disparity.
D)
a permitted disparity of 20% and excess benefit of 40%.

A

D

A flat defined benefit plan could provide the LESSER OF the 20% base benefit percentage or 26.25%, and excess benefit of 20% + 20%.

How well did you know this?
1
Not at all
2
3
4
5
Perfectly
29
Q

Juan worked for ABC for 23 years. DP plan pays 2% of employees average of his five highest years of income. Juan’s average is $60k. How much will he receive each month if he retires this year?

$800
$3,000
$2,300
$1,200

A

C

$60,000 * .02 * 23 (years) = yearly pension income
/12 = 2,300 monthly income

How well did you know this?
1
Not at all
2
3
4
5
Perfectly
30
Q

Gary is employed by the city of Great Rapids, and his sister, Julie, works for Big Heart Children’s Home, a 501(c)(3) nonprofit. Gary participates in a 457 plan and Julie participates in a Section 403(b) plan.

Which of the following statements is CORRECT in indicating how their respective plans compare to each other?

Both plans are based on contracts with the employer.
Both plans allow for the deferral of salary into the plans.
Neither plan is a qualified plan.
Both plans are subject to limits on the amount that can be contributed to the plan.
A)
I and II
B)
I and III
C)
I, II, III, and IV
D)
II, and IV

A

All

How well did you know this?
1
Not at all
2
3
4
5
Perfectly
31
Q

Which of the following describe differences between a tax-advantaged retirement plan and a qualified plan?

IRA-funded employer-sponsored tax-advantaged plans may not incorporate loan provisions.
Employer stock distributions from a tax-advantaged plan do not benefit from NUA tax treatment.

A

Both

IRA funded employer sponsored tax advantage plans are SEPs, SARSEPs, and SIMPLE IRAs.

How well did you know this?
1
Not at all
2
3
4
5
Perfectly
32
Q

Which of the following would NOT be a permitted disparity for a defined benefit plan that uses Social Security integration?

A)
An excess benefit percentage of 20%, if the base percentage is 15%
B)
An excess benefit percentage of 60%, if the base percentage is 30%
C)
An excess benefit percentage of 10% if the base percentage is 5%
D)
An excess benefit percentage of 40%, if the base percentage is 20%

A

B

Base percentage + permitted disparity = excess benefit %
Permitted disparity goes up to a max of 26.25%.

26.25% = 0.75% per year up to 35 years.

How well did you know this?
1
Not at all
2
3
4
5
Perfectly
33
Q

Worthy Tools Inc. is designing a money purchase pension plan for eligible employees. The following information relates to Worthy Tools Inc.:

The seven key employees, all with more than eight years of service, will be eligible for the plan.
Of the 24 full-time rank-and-file employees, ranging in age from 19 to 61 years old, 22 will be eligible for the plan.
Turnover is high; the rank-and-file employees’ average tenure is 20 months. Although occasionally a nonkey employee stays at the company for over two years, none has ever completed more than 2.5 years of service.
The key employees’ compensation totals $650,000; the plan’s covered payroll will total $1,000,000.
The company is installing the plan primarily to provide for its officers’ retirement benefits and is not particularly concerned about employee turnover or about providing retirement benefits for employees. In fact, the company would prefer to minimize the benefits available to rank-and-file employees.
Based on the information about Worthy Tools, which one of the following vesting schedules would be appropriate for the company pension plan?

A)
5-year cliff
B)
3- to 7-year graded
C)
2- to 6-year graded
D)
3-year cliff

A

3 Year Cliff

The plan will be top-heavy since the pay of key employees is 65% (more than 60%) of the total pay of all plan participants. (Employer contributions to a money purchase plan are allocated to participants’ accounts based on relative pay.) The maximum vesting schedules available to a defined contribution plan are 3-year cliff and 2- to 6-year graded; 3-year cliff would be the best choice for minimizing benefits to rank-and-file employees, who have historically stayed a maximum of two years with the company.

LO 1.3.1

How well did you know this?
1
Not at all
2
3
4
5
Perfectly
34
Q

Benjamin Scott, age 42, earns $110,000 a year and wants to establish a profit-sharing plan. He employs four people whose combined salaries are $58,000 and who range in age from 24 to 30. The average employment period for all employees is three-and-a-half years. Which vesting schedule is best suited for Benjamin’s plan?

A)
Three-to seven-year graded vesting
B)
Five-year cliff vesting
C)
Two-to-six-year graded vesting
D)
Three-year cliff vesting

A

The choices for vesting in a defined contribution plan are: (1) 100% full and immediate, (2) three-year cliff, or (3) two-to-six graded. Because of the average length of employment, the most suitable vesting schedule from Benjamin’s point of view (cash flow if termination occurs and reallocated forfeitures to Benjamin) is graded vesting. Five-year cliff vesting and three-to-seven-year graded vesting are not available to Benjamin’s company for a profit-sharing plan.

LO 1.3.1

How well did you know this?
1
Not at all
2
3
4
5
Perfectly
35
Q

Which of the following statements regarding nonqualified retirement plans is CORRECT?

A)
Benefits are received by the employee income-tax-free.
B)
Contributions are deductible by the employer when contributed to the plan.
C)
Nonqualified plans are subject to the same Employee Retirement Income Security Act of 1974 (ERISA) requirements as qualified plans.
D)
Nonqualified plans do not have to meet the nondiscrimination requirements that apply to qualified plans.

A

Nonqualified plans do not have to meet the nondiscrimination requirements that apply to qualified plans. Nonqualified plans are not subject to all ERISA requirements applicable to qualified plans. Benefits are not deductible by the employer until paid and are includable in the employee’s taxable income at the time of receipt.

LO 1.2.1

How well did you know this?
1
Not at all
2
3
4
5
Perfectly
36
Q

Which of the following qualified plan reporting and disclosure documents is considered the annual report for the plan, and must be filed annually with the IRS?

A)
Form 5500
B)
Summary Annual Report (SAR)
C)
Summary of Material Modification (SMM)
D)
Summary Plan Description (SPD)

A

An annual report (Form 5500 series) must be filed with the IRS annually by the end of the seventh month after the plan year ends. The Form 5500 is also required to be filed with the Department of Labor.

LO 1.4.1

How well did you know this?
1
Not at all
2
3
4
5
Perfectly
37
Q

What is the 2023 limit on maximum includible compensation for retirement plan purposes?

A)
$330,000
B)
$66,000
C)
$22,500
D)
$265,000

A

For the purposes of determining qualified plan benefits and employer contributions, employee includible compensation is capped at $330,000 for 2023.

LO 1.3.2

How well did you know this?
1
Not at all
2
3
4
5
Perfectly
38
Q

Which of these is FALSE regarding defined contribution plans?

A)
Includible compensation is limited to the lesser of 100% of compensation or $265,000 in 2023.
B)
The employer contribution limit is 25% of the participating employees’ payroll.
C)
The retirement benefit is not certain; investment risk is borne by the participant.
D)
The maximum allowable employee deferral amount for workers is $22,500 in 2023, not counting any catch-ups.

A

The maximum amount of includible compensation is $330,000, not $265,000. $265,000 is a maximum defined benefit test number in 2023.

LO 1.3.2

How well did you know this?
1
Not at all
2
3
4
5
Perfectly
39
Q

Which of the following are CORRECT statements about the overall limits on employer contributions to and deductions for qualified plan contributions?

An employer’s deduction for contributions to a money purchase pension plan and profit sharing plan cannot exceed 25% of the participants’ payroll.
If a company has a defined benefit plan and a defined contribution plan and no employee is covered by both plans, the overall deduction limit does not apply.
An employer’s deduction for contributions to a defined benefit pension plan and profit sharing plan cannot exceed the lesser of the amount necessary to satisfy the minimum funding standards or 25% of the participants’ payroll.
A plan may lose its qualified status if the contribution for a participant exceeds IRC Section 415 limits.
A)
II and IV
B)
I, II, and IV
C)
I and II
D)
I and III

A

Options I, II, and IV accurately state the following rules: the deduction limitation for defined contribution pension and profit sharing plans; one of the rules governing the contribution limit for an employer who maintains both a defined benefit and a defined contribution plan; and the possible loss of qualified status if the contribution for a plan participant exceeds the Section 415 limits. Option III is incorrect because if the overall deduction limit applies, the deduction for a combination defined benefit plan and profit sharing plan cannot exceed the greater of the amount necessary to meet the minimum funding requirements or 25% of the participants’ payroll.

LO 1.3.2

How well did you know this?
1
Not at all
2
3
4
5
Perfectly
40
Q

Which of the following is CORRECT regarding IRS Form 5500?

A)
The IRS 5500 is known as the employer’s annual return/report to the IRS of an employee benefit plan.
B)
All of these statements are correct.
C)
A simplified version, Form 5500-EZ, is available for certain small employers.
D)
Filing an IRS 5500 is an ERISA requirement.

A

b

41
Q

Joe has worked for XYZ Co. for 30 years and is a participant in his employer’s traditional defined benefit pension plan. He is retiring this year. The plan formula provides a pension equal to the average of the participant’s final three years of compensation. Joe’s final three years of compensation were $280,000, $290,000, and $340,000.

What will be the amount of Joe’s pension under the plan in 2023?

A)
$221,000
B)
$265,000
C)
$300,000
D)
$303,333

A

Applying the 2023 covered compensation limit of $330,000, Joe’s final three years of compensation averages to $300,000. However, the maximum defined benefit pension payable from a traditional defined benefit pension plan is $265,000 (2023).

LO 1.3.2

42
Q

Which of the following is NOT required to be distributed to qualified plan participants on an annual basis?

Summary Plan Description (SPD)
Form 5500
Summary Annual Report (SAR)
Summary of Material Modification (SMM)
A)
IV only
B)
I and II
C)
I, II, and IV
D)
III and IV

A

c

Only a SAR, summarizing the basic information included in the Form 5500 series, must be provided to plan participants each year within nine months of the end of the plan year. An SPD must be provided automatically to all plan participants within 120 days after the plan is established or 90 days after a new participant enters an existing plan, not annually. Form 5500 must be filed with the IRS and DOL annually, but is not required to be provided annually to plan participants; however, participants have a right to see the full Form 5500 if needed and requested. An SMM, explaining any substantive changes that occurred to the SPD within the past year, must be issued as needed.

43
Q

Velvet Lawns, Inc., employs 26 full-time workers and provides a money purchase pension plan for eligible employees. All 26 employees are plan participants this year. Jack, the owner of the company, has an account balance of $134,000. The total of the account balances of all plan participants amounts to $215,000.

Which of the following statements apply to this plan?

The plan would not pass the required coverage test and is therefore discriminatory.
The plan is top heavy.
The plan may use either a five-year cliff or three- to seven-year graded vesting schedule.
The plan must comply with requirements for minimum contributions to non-key employees.
A)
I and III
B)
II, III, and IV
C)
II and IV
D)
I, II, and IV

A

II IV

If all 26 employees are participating in the plan, the coverage and nondiscrimination tests would be passed. The plan would, however, be top heavy (Jack’s $134,000 account balance is 62% of the $215,000 total of all account balances). Top-heavy plans must comply with minimum contribution requirements for non-key employees and use a top-heavy vesting schedule. All defined contribution plans are now required to vest no less rapidly than three-year cliff or two- to six-year graded.

LO 1.3.1

44
Q

Which of the following is an Employee Retirement Income Security Act of 1974 (ERISA) requirement that qualified plans must meet?

Coverage
Participation
Vesting
Reporting and disclosure
A)
I and II
B)
IV only
C)
I, II, III, and IV
D)
II, III, and IV

A

All four, and fiduciary requirements (CFP or not as you remember from another question)

45
Q

Jeff wants to establish a qualified plan for his business to provide employees of the company with the ability to save for retirement. Which of the following plans is a qualified plan?

Profit-sharing plan
Simplified employee pension (SEP) plan
SIMPLE IRA
Top-Hat plan
A)
II and III
B)
I, II, III, and IV
C)
IV only
D)
I only

A

D I only

Only the profit sharing plan is a qualified plan.

SIMPLE and SEP IRAs are tax advantaged plans and section 457 is a Nonqualified plan.

46
Q

Which of the following are minimum coverage tests for qualified retirement plans?

Active participation test
Average benefits percentage test
Ratio test
Maximum compensation test
A)
I and II
B)
I, II, and III
C)
II, III, and IV
D)
II and III

A

II III

For QUALIFIED plans
MINIMUM COVERAGE
Just memorize: Average Benefits % test, Ratio test

The two minimum coverage tests for qualified retirement plans are the average benefits percentage test and the ratio test. To be qualified, a retirement plan must meet at least one of these tests if the plan does not meet the percentage (safe harbor) test.

47
Q

Which of these are CORRECT statements about the top-heavy rules?

An employer’s minimum top-heavy contribution to a money purchase pension plan is the lesser of 3% of compensation per year for non-key employees or the highest contribution percentage made for a key employee if the highest contribution percentage is less than 3% of compensation.
An employer’s top-heavy contribution to a money purchase pension plan must always be at least 3% of compensation per year for each employee.
For purposes of applying the 60% top-heavy test, benefits include any distributions made due to separation from service during the last year and any in-service distributions made during the last five years.
Any employee with annual earnings from the employer in excess of $330,000 is considered a key employee in 2023.
A)
I, II, and IV
B)
I and III
C)
I and II
D)
III and IV

A

Options I and III are correct statements about the minimum contribution to a top-heavy plan and the years taken into consideration for purposes of determining the top-heavy status of a plan. Option II is incorrect because the required minimum top-heavy contribution must be made on behalf of each non-key employee. Also, if the contributions for key employees are less than 3%, then the contributions for non-key employees can also be less than 3%. Option IV is incorrect because a key employee is defined as a more-than-1% owner of the employer company, with annual compensation from the employer in excess of $150,000 or an officer of the employer whose annual compensation from the employer exceeds $215,000 in 2023, or a more-than-5% owner.

LO 1.3.1

48
Q

ERISA requirements for qualified plans include

coverage and vesting.
fiduciary requirements.
reporting and disclosure.
participation requirements.
A)
I, II, and III
B)
II and IV
C)
I and III
D)
I, II, III, and IV

A

ALL

49
Q

Which of the following are CORRECT statements about the legal requirements for pension plans subject to Pension Benefit Guaranty Corporation (PBGC) coverage?

The PBGC may initiate a standard termination proceeding if the pension plan is unable to pay benefits when due.
PBGC-guaranteed benefits exclude medical insurance benefits, benefits in excess of the PBGC limit, and lump-sum benefit payments.
The amendment of a defined benefit pension plan into a money purchase pension plan will result in termination of the defined benefit plan.
Pension plans are required to pay premiums to the PBGC.
A)
I, II, and III
B)
I, II, and IV
C)
I and II
D)
II and III

A

Options II and III are correct statements. Option I is incorrect because the PBGC may initiate an involuntary distress termination proceeding if a pension plan is unable to pay pension benefits when they become due. Option IV is incorrect because only defined benefit plans are generally required to pay PBGC premiums; “pension” plans include some defined contribution plans. In this case, a “pension plan” is a category, not a single type of plan. Pension plans require mandatory annual funding each and every year. The other category besides pension plans is called “profit sharing plans.” In this usage, a profit sharing plan is a plan that does not require mandatory annual contributions. You can remember the pension plans as the “Be my cash target plans”: “B” for benefit in defined benefit plans, “M” for money purchase plans, “Cash” for cash balance plans, and “Target” for target benefit plans. This mnemonic device not only separates the pension plans and profit sharing plans, but it can be extremely helpful in selecting retirement plans for a business. For example, if a firm cannot commit to mandatory annual funding, then the “Be my cash target plans” are eliminated.

50
Q

The purpose of integrating a qualified retirement plan with Social Security is to

A)
exclude employees from participating in the plan if their compensation exceeds the Social Security taxable wage base.
B)
reduce plan contributions for employees who are covered by Social Security.
C)
allow a higher contribution rate for highly compensated employees than for nonhighly compensated employees.
D)
permit the use of longer vesting schedules.

A

C

Social Security effectively discriminates against highly compensated employees because earnings over the taxable wage base are not counted for purposes of accruing benefits. Integrated plans are designed to help remedy this inequity by allowing a higher contribution rate for highly compensated employees.

LO 1.3.3

51
Q

Annual additions to qualified retirement plans include which of these?

Employer contributions
Employee contributions
Interest and dividend income
Forfeitures reallocated to plan participants
A)
I, II, and IV
B)
I and II
C)
I, II, and III
D)
II, III, and IV

A

A

Only Statement III is incorrect. Investment earnings, such as interest and dividends, are not included as annual additions.

52
Q

In which situation(s) can the Pension Benefit Guaranty Corporation (PBGC) terminate a defined benefit pension plan?

The PBGC does not have the authority to terminate an employer’s defined benefit pension plan.
Joe’s Shoes Inc. has not met the minimum funding standards for the corporation’s defined benefit pension plan.
Ann has retired from her position and her former employer’s defined benefit pension plan cannot pay her retirement benefit because it lacks the funds.
A professional service corporation with 15 active participants cannot meet the minimum funding standards of the defined benefit pension plan and has previously paid no PBGC premiums.
A)
II and III
B)
II, III, and IV
C)
I only
D)
III only

A

Statements II and III are correct. The PBGC can terminate a defined benefit plan if:

minimum funding standards are not met;
benefits cannot be paid when due; and
the long-run liability of the company to the PBGC is expected to increase unreasonably.
A professional service corporation with 25 or fewer participants is not required to maintain PBGC coverage.

LO 1.1.1

53
Q

Which of the following statements regarding qualified retirement plans is CORRECT?

Money purchase pension plans, employee stock ownership plans (ESOP), target benefit pension plans, and stock bonus plans are all examples of qualified retirement plans.
Top-hat plans and cash balance pension plans are examples of qualified retirement plans.
A)
Both I and II
B)
I only
C)
Neither I nor II
D)
II only

A

b

Qualified Plans in this question:
MPPP, ESOP, TBPP, Stock bonus plans, CBPP

NonQualified Plans in this question:
Top-Hat plans

54
Q

The qualified retirement plan maintained by the ABC Corporation imposes a two-year waiting period before new employees may enter the plan. Which of the following statements is CORRECT?

A)
The plan is a Section 401(k) plan.
B)
The plan must provide 100% vesting to all employees immediately upon entry.
C)
The plan is in violation of ERISA.
D)
The plan may use the normal qualified plan vesting schedules.

A

B

Reminder: it’s different from a 2-6 yr graded vestment schedule.
2 year waiting period. After that waiting period, they get full rights (vesting) to keep all their retirement benefits (100% vesting) after they leave the company.
2-6 years graded. Even though after 2 years, you begin to get vestment, there may not be a waiting period at all, and you can start to make contributions (with possible employer matching). You will only begin to have the right to keep certain incremental percentages of your retirement benefits if you leave early (depending on how many years you stayed with the company).

Most types of qualified plans are allowed to increase the waiting period to two years of service, but plans adopting this rule must provide 100% vesting to all employees immediately upon entry. Section 401(k) plans cannot require a two-year waiting period.

LO 1.3.1

55
Q

Which of the following statements regarding Social Security integration and defined contribution plans is CORRECT?

The integration level can be less than the taxable Social Security wage base.
The maximum permitted disparity will depend on whether the integration level is equal to the taxable wage base or below it.
Integration can be used to enhance an owner’s contribution to the plan if the owner’s compensation is in excess of the Social Security wage base.
The integration level cannot be greater than the Social Security taxable wage base.
A)
II, II, and IV
B)
I and IV
C)
I, II, III, and IV
D)
I, II, and III

A

If the integration level is less than the taxable wage base, the permitted disparity amount may be reduced. Integration can enhance an owner’s and/or key employee’s contribution rate if the compensation is in excess of the Social Security wage base. A defined contribution plan cannot have integration levels greater than the taxable wage base.

56
Q

In a profit-sharing plan that is integrated with Social Security and uses a 4% base contribution rate, what is the maximum excess contribution percentage that may be applied?

A)
5.5%
B)
5.7%
C)
9.7%
D)
8.0%

A

Under the permitted disparity rules, the maximum permitted excess contribution percentage is the lesser of two times the base percentage; and the base percentage plus 5.7%, resulting in a total excess contribution percentage of 8.0% (2 × 4%).

LESSER OF: 2*%
OR
%+5.7%

57
Q

In the ratio test used to determine whether a qualified plan is nondiscriminatory, what is the minimum percentage of nonhighly compensated employees who must be covered as compared to the percentage of covered highly compensated employees?

A)
75%
B)
60%
C)
70%
D)
51%

A

A qualified plan will pass the ratio test if the percentage of nonhighly compensated employees covered under the plan is at least 70% of the percentage of highly compensated employees who are covered.

58
Q

The maximum service requirement that a thrift plan may impose as a condition of participation is

A)
one year.
B)
one-and-a-half years.
C)
two years.
D)
six months.

A

By law, the maximum service requirement that a thrift plan may impose is two years of service. An employee must become a participant in a thrift plan on the next plan entry date following the later of attainment of age 21 or the completion of two years of service. Of course, a more liberal participation requirement may be imposed. The maximum for a 401(k) is one year.

LO 1.3.1

59
Q

Max is the finance director for Bland Foods, Inc. He is trying to implement a new qualified retirement plan for the company. There are numerous federal guidelines with which the company must comply. Which of the following federal agencies is tasked with supervising the creation of new, qualified retirement plans?

A)
ERISA
B)
PBGC
C)
FINRA
D)
IRS

A

IRS

60
Q

Which of these is not an example of a qualified retirement plan?

A)
Traditional profit-sharing plan
B)
Section 403(b) plan
C)
Section 401(k) plan
D)
Employee stock ownership plan (ESOP)

A

b

The answer is Section 403(b) plan. A Section 403(b) plan is a tax-advantaged plan but not an ERISA-qualified retirement plan. While tax-advantaged plans are very similar to qualified plans, there are some minor differences. For example, a tax-advantaged plan is not allowed to have net unrealized appreciation (NUA) treatment. These plans are also not allowed to offer 10-year forward averaging or special pre-1974 capital gains treatment. Tax-advantaged plans also have less restrictive nondiscrimination rules. Otherwise they are very similar to qualified plans.

LO 1.2.1

61
Q

What is the penalty for a prohibited transaction?

A)
50%
B)
15%
C)
10%
D)
25%

A

b

62
Q

What is the IRS form that also serves as a qualified plan’s annual financial report?

A)
Form 1120
B)
Form 5500
C)
Form 1040
D)
Form 5200

A

b

A plan’s annual financial report is filed on Form 5500 and must be filed with the IRS within seven months after the close of each plan year. The report is designed to provide a complete disclosure of all financial information relevant to plan operation.

LO 1.4.1

63
Q

Which of the following is NOT an example of a qualified retirement plan?

A)
Section 401(k) plan
B)
New comparability plan
C)
Section 403(b) plan
D)
Employee stock ownership plan (ESOP)

A

The answer is a Section 403(b) plan is a tax-advantaged plan but not an ERISA-qualified retirement plan. While tax-advantaged plans are very similar to qualified plans, there are some minor differences.

LO 1.2.1

64
Q

Susan is the fiduciary of the SLH retirement plan. The entity responsible for monitoring her actions as a fiduciary is

A)
the SPD.
B)
the ERISA.
C)
the DOL.
D)
the PBGC.

A

The Department of Labor (DOL) governs the actions of plan fiduciaries and ensures compliance with the ERISA plan reporting and disclosure requirements.

LO 1.1.1

65
Q

Which of these is a prohibited transaction?

A)
The plan’s acquisition of employer securities or real property in excess of legal limits
B)
The transfer of any plan assets or use of plan assets for the benefit of a party in interest
C)
The sale, exchange, or lease of any property between the plan and a party in interest
D)
All of these are prohibited transactions

A

All of these are prohibited transactions.

LO 1.4.1

66
Q

Angela has received a sizable inheritance. She makes $12,000 a year working part time. She has been deferring 5% into her 401(k) at work even though there is no match. What is the maximum she can defer into the 401(k) in 2023?

A)
$6,500
B)
$12,000
C)
66,000
D)
$22,500

A

B

The maximum annual addition for 2023 is the lesser of 100% of compensation, or $66,000. Thus, Angela is limited to $12,000.

LO 1.3.2

Compensation this year: $12,000

67
Q

Which of the following plans allow the excess method of permitted disparity?

Money purchase pension plans
Defined benefit pension plans
Employee stock ownership plan (ESOP)
Simplified employee pension (SEP)
A)
I, II, and III
B)
I, II, and IV
C)
I, III, and IV
D)
II, III, and IV

A

I II IV

all plans are allowed to integrate with Social Security except ESOPs, SIMPLEs, and SARSEPs. The excess method is allowed for all plans allowing integration, whereas the offset method is only allowed for defined benefit pension plans.

68
Q

Which of the following statements regarding permitted disparity rules as they relate to qualified retirement plans are CORRECT?

A defined benefit pension plan using the permitted disparity rules may be an excess method plan.
A defined benefit pension plan using the permitted disparity rules may be an offset method plan.
A defined contribution plan using the permitted disparity rules may be an excess method plan.
A defined contribution plan using the permitted disparity rules may be an offset method plan.
A)
II, III, and IV
B)
I and IV
C)
I, II, III, and IV
D)
I, II, and III

A

I II III

A defined contribution plan can satisfy the permitted disparity (integration) rules only if it uses an excess method plan integration formula. Therefore, Statement IV is incorrect. Defined contribution plans cannot use an offset method plan formula because they have no promised benefit formula. Defined benefit pension plans can use either an offset or an excess method plan formula for integration.

LO 1.3.3

69
Q

Peter’s employer maintains a qualified defined benefit pension plan. There are 100 eligible employees working for the company. What is the minimum number of employees the retirement plan must cover to satisfy the 50/40 test?

A)
70
B)
40
C)
50
D)
90

A

40

Sparknotes for below:
50 people / 40% of “employed humans”
No key employees or highly compensated or anything like that.

Under the 50/40 test, a defined benefit plan must cover the lesser of 50 employees or 40% of all eligible employees. In this case, the lesser of 50 employees or 40% of all eligible employees (100) is 40 employees. One way to remember the 50/40 test is the phrase people before percentages (50 people or 40%). Also, note that there are no qualifiers to the types of people. It is not 50 non-highly compensated people. It is just 50 humans who work for the employer.

LO 1.3.1

70
Q

Which of the following statements regarding top-heavy plans is CORRECT?

An accelerated vesting schedule is used when a defined benefit pension plan is top heavy.
A qualified plan is considered top heavy if it provides more than 50% of its aggregate accrued benefits or account balances to key employees.
Top-heavy defined benefit plans must provide a minimum benefit of 2% per year of service for up to 10 years (20%) for all non-key employees.
For a top-heavy plan, a key employee is an employee who owns more than 3% of the employer with compensation greater than $135,000 (2023).
A)
II and IV
B)
I and III
C)
II and III
D)
I and II

A

I III

An accelerated vesting schedule is used when a defined pension benefit plan is top heavy. A defined contribution plan always requires an accelerated vesting schedule. A qualified plan is considered top heavy if it provides more than 60% of its aggregate accrued benefits or account balances to key employees. A key employee is an employee who, at any time during the plan year, is the following: greater than a 5% owner; a greater than 1% owner with compensation greater than $150,000 (not indexed); or an officer of the employer with compensation greater than $215,000 in 2023.

LO 1.3.1

71
Q

Qualified retirement plans have which of the following characteristics?

They are subject to ERISA requirements.
They offer tax-deferred earnings to employees.
They can discriminate in favor of highly compensated employees.
They provide a tax deduction for the employer only when the employee is income taxed on the money.
A)
I and II
B)
I, II, and III
C)
III and IV
D)
I, II, III, and IV

A

Statements I and II are correct. Qualified retirement plans are subject to ERISA requirements and provide tax deferral on investment earnings for employees. While qualified plans in general can provide different levels of benefits to different classes of employees, qualified plans cannot “discriminate in favor of highly compensated employees” in the sense that there is a legal limit to the amount of the difference. As long as the difference is inside the legal limits, the plan is not discriminatory (by definition). Qualified retirement plans provide an immediate tax deduction on employer contributions.

72
Q

Which of these statements regarding top-heavy plans is CORRECT?

An accelerated vesting schedule is used when a defined benefit pension plan is top heavy.
A qualified plan is considered top heavy if it provides more than 75% of its aggregate accrued benefits or account balances to key employees.
Top-heavy defined benefit plans must provide a minimum benefit of 2% per year of service for up to 10 years (20%) for all non-key employees.
For a top-heavy plan, a key employee is an employee who owns more than 3% of the employer with compensation greater than $140,000 (2023).
A)
I and III
B)
II and IV
C)
II and III
D)
I and II

A

An accelerated vesting schedule is used when a defined pension benefit plan is top heavy. A defined contribution plan always requires an accelerated vesting schedule. A qualified plan is considered top heavy if it provides more than 60% of its aggregate accrued benefits or account balances to key employees. A key employee is an employee who, at any time during the plan year, is the following: greater than a 5% owner; a greater than 1% owner with compensation greater than $150,000 (not indexed); or an officer of the employer with compensation greater than $215,000 in 2023.

LO 1.3.1

73
Q

Paul’s employer maintains a qualified defined benefit pension plan. There are 100 eligible employees working for the company. What is the minimum number of employees the retirement plan must cover to satisfy the 50/40 test?

A)
90
B)
40
C)
70
D)
50

A

b

Under the 50/40 test, a defined benefit plan must cover the lesser of 50 employees or 40% of all eligible employees. In this case, the lesser of 50 employees or 40% of all eligible employees (100) is 40 employees. One way to remember the 50/40 test is the phrase people before percentages (50 people or 40%). Also, note that there are no qualifiers to the types of people. It is not 50 non-highly compensated people. It is just 50 humans who work for the employer.

LO 1.3.1

74
Q

Ryan, who is 50, is employed by Best Mutual Funds (BMF) and participates in its profit sharing Section 401(k) plan for 2023. The plan allocates contributions based on relative compensation and is not integrated with Social Security. Ryan’s current annual compensation is $80,000, and he has elected to defer 5% of compensation into the Section 401(k) plan. Including Ryan, 35 employees with a total covered payroll of $1.8 million participate in the plan and have elected to defer a total of $72,000 (4% of payroll). BMF’s matching contributions to the plan total $54,000 (3% of total covered payroll). What is the limit on annual additions to Ryan’s individual account?

A)
$66,000
B)
$15,500
C)
$30,000
D)
$22,500

A

a

The limit on annual additions to Ryan’s account in 2023 is the lesser of 100% of compensation ($80,000) or $66,000.

LO 1.3.2

75
Q

A qualified plan must allow a 22-year-old employee who satisfies the plan’s one-year service requirement to commence participation in the plan no later than which of the following events?

The first day of the plan year beginning after the date on which the employee satisfies such requirements
The last day of the plan year during which the employee satisfies such requirements
The date six months after the date on which the employee satisfies such requirements
The first day of the plan year in which the employee satisfies such requirements
A)
I and III
B)
I and II
C)
III and IV
D)
II and IV

A

a

A plan that contains an age 21 and one year of service requirement must provide that an employee who meets the age and service requirements shall participate in the plan no later than the earlier of the first day of the first plan year beginning after the date on which the employee satisfies such requirements, or the date six months after the date by which the employee satisfies such requirements. Notice that the longest an eligible employee would have to wait is six months.

LO 1.3.1

76
Q

What are the requirements and effect of an eligible investment advice arrangement under the Pension Protection Act?

An eligible investment advice arrangement allows a plan fiduciary to give advice, including recommending her own proprietary funds without violating fiduciary rules.
The investment advisor’s fees must be neutral.
An unbiased computer model certified by an independent expert to create a recommended portfolio for the client’s consideration is used.
Investment advisors for IRAs may only use the unbiased computer model option when providing eligible investment advice.
A)
II and III
B)
I, II, and III
C)
I and II
D)
I, II, III, and IV

A

I II III

Investment advisors for IRAs may only use the neutral fee option and not the computer model option when providing eligible investment advice.

LO 1.4.1

77
Q

Which of the following statements regarding a top-heavy plan is CORRECT?

A top-heavy plan is one that provides more than 40% of its aggregate accrued benefits or account balances to key employees.
A top-heavy defined pension benefit plan must provide a minimum benefit accrual of 2% multiplied by the number of years of service (up to 20%).
For a top-heavy defined contribution plan, the employer must make a minimal contribution of 3% of annual covered compensation for each eligible non-key employee. If the contribution percentage for key employees is less than 3%, the contribution percentage to non-key employees can be equal to the key employees’ percentage.
A top-heavy defined benefit pension plan must provide accelerated vesting.
A)
II, III, and IV
B)
I, II, III, and IV
C)
I, II, and IV
D)
I, II, and III

A

II III IV

Only Statement I is incorrect. A top-heavy plan is one that provides more than 60% of its aggregate accrued benefits or account balances to key employees.

LO 1.3.1

78
Q

The Smith Corporation has a profit-sharing plan with a 401(k) provision. The company matches dollar-for-dollar up to 5%. Pedro makes $150,000 and defers 5% into the 401(k) for 2023. The Smith Corporation has had a banner year and is considering a large contribution to the profit-sharing plan. What is the most that could be contributed to Pedro’s profit-sharing account this year?

A)
$58,500
B)
$22,500
C)
$51,000
D)
$66,000

A

c

The maximum allowed contribution for 2023 is $451,000. The section annual additions limit for 2023 is $66,000. However, Pedro has already contributed $7,500, and this amount has been matched. Thus, $15,000 has already gone toward the $66,000 annual additions limit for 2023.

LO 1.3.2

79
Q

Which of the following describe differences between tax-advantaged retirement plans and qualified plans?

IRA-funded employer-sponsored tax-advantaged plans may not incorporate loan provisions.
Employer stock distributions from a tax-advantaged plan do not benefit from NUA tax treatment.
A)
II only
B)
I only
C)
Neither I nor II
D)
Both I and II

A

Both statements are correct. IRA-funded employer-sponsored tax-advantaged plans are SEPs, SARSEPs, and SIMPLE IRAs.

LO 1.2.1

For Tax advantaged non qualified, think 403(b)

403(b) plans don’t get NUA, loan provisions.

80
Q

Which of these statements regarding Social Security plan integration is false?

Because there is a disparity in the Social Security system, all retirement plans are allowed to integrate with Social Security.
Only the excess method can be used by a defined benefit pension plan.
Under the offset method of integration, a fixed or formula amount reduces the plan formula.
The maximum increase in benefits for earnings above covered compensation level is 5.7% for a defined benefit pension plan.
A)
II and IV
B)
I and III
C)
I, II, and IV
D)
II and III

A

C I II IV

Statement I is incorrect because not all retirement plans are allowed to integrate with Social Security. For example, ESOPs and SARSEPs are not permitted to use integration. Statement II is incorrect because the defined benefit pension plan can use either the excess or offset method in integrating with Social Security. Only the excess method can be used with defined contribution plans. Statement IV is incorrect because the permitted disparity limit for a defined benefit pension plan is 26.25% above the covered compensation level (0.75% per year for up to 35 years).

LO 1.3.3

81
Q

Which of the following persons is a party-in-interest for a qualified plan?

An officer of the pension plan
The sponsor company
Bill, who owns 60% of XYZ, the corporate sponsor
Mary, a CFP® professional, the investment adviser to the plan
A)
I and IV
B)
I and III
C)
II, III, and IV
D)
I, II, III, and IV

A

All of those listed are parties in interest. The rule for statement III is ownership of 50% or more of the corporate sponsor.

82
Q

Mike Robertson is an employee of Tower Inc. and participates in the corporation’s profit sharing plan. Mike owns 10% of Tower’s voting stock but is not an officer of the corporation. John Tower owns the rest of the corporation’s voting stock. The following schedule summarizes Mike’s annual earnings from Tower Inc.

Year Annual Earnings
Year before last $50,000
Last year $86,000
This year $115,000
Is Mike a key employee of Tower Inc.? Why or why not?

A)
Yes, because Mike’s 10% ownership of Tower Inc. makes him a greater-than-5% owner.
B)
No, because Mike’s compensation is less than the dollar limitation in effect under IRC Section 415(b)I(A).
C)
No, because Mike’s compensation is less than $150,000.
D)
Yes, because Mike owns one of the largest interests in Tower Inc.

A

a

An individual who owns more than 5% of the employer company is treated as a key employee. Mike does not satisfy one of the other definitions of a key employee:

An officer of the employer whose annual compensation from the employer exceeds $215,000 in 2023
A more-than-1% owner of the employer whose annual compensation from the employer exceeds $150,000
LO 1.3.1

83
Q

Big Bucks Bank, as the plan trustee for the XYZ Corporation profit-sharing plan, has entered into a loan with the plan secured by the individual account balances of the plan participants. What has just occurred?

A)
A disqualified loan
B)
A financial obligation incurred in the ordinary course of business
C)
A contribution to the plan consistent with the annual additions limit
D)
A prohibited transaction

A

This is an example of a prohibited transaction under ERISA and the Internal Revenue Code. The lending of money between a qualified plan and a disqualified person, such as the plan trustee/bank here, is prohibited and subject to an initial 15% prohibited transaction penalty. If the transaction is not corrected, there is an additional 100% penalty for the amount of the prohibited transaction. Also, the party at interest is personally liable for the penalty if the plan assets are not sufficient to pay the penalty.

LO 1.4.1

84
Q

An employer’s annual contributions to a qualified defined contribution plan must be within statutory limits regarding the allowable amount. An employer’s annual contribution to a defined contribution plan is limited to ____% of the firm’s covered payroll.

A)
10
B)
20
C)
25
D)
15

A

25

85
Q

Roderick Manufacturing maintains a qualified defined benefit plan. There are 100 eligible employees working for the company. What is the minimum number of employees the retirement plan must cover to satisfy the 50/40 test?

A)
40
B)
90
C)
70
D)
50

A

The answer is 40. Under the 50/40 test, a defined benefit plan must cover the lesser of 50 employees or 40% of all eligible employees. In this case, the lesser of 50 employees or 40% of all eligible employees (100) is 40 employees. One way to remember the 50/40 test is the phrase “people before percentages” (50 people or 40%). Also, note that there are no qualifiers to the types of people. It is not 50 non-highly compensated people. It is just 50 employees who work for Roderick Manufacturing.

LO 1.3.1

85
Q

Max wants to establish a qualified plan for his business to provide employees of the company the ability to save for retirement. Which of the following plans is a qualified plan?

Defined benefit pension plan
SIMPLE IRA
Section 457 plan
Profit sharing plan
A)
I and IV
B)
II and III
C)
I, II, III and IV
D)
IV only

A

I IV

-Of the plans listed, only the defined benefit plan and the profit sharing plan are qualified plans (statements I and IV). The SIMPLE IRA is a tax-advantaged plan, and the Section 457 plan is a nonqualified plan.

LO 1.2.1

85
Q

Dr. Bennett has established a top-heavy profit-sharing plan for himself and his employees. He made a 1% of compensation contribution to all participants’ accounts this plan year. Which of these statements is CORRECT?

A)
He must add another 1.25% to non-key employee accounts.
B)
He must add another 0.25% to non-key employee accounts.
C)
No further contribution is required.
D)
He must contribute another 1% to each employee.

A

c

Dr. Bennett does not have to make any further contributions because both non-key and key employees received the same percentage contribution.

LO 1.3.1

85
Q

Ryan, who is 50, is employed by Best Mutual Funds (BMF) and participates in its profit sharing Section 401(k) plan for 2023. The plan allocates contributions based on relative compensation and is not integrated with Social Security. Ryan’s current annual compensation is $80,000, and he has elected to defer 5% of compensation into the Section 401(k) plan. Including Ryan, 35 employees with a total covered payroll of $1.8 million participate in the plan and have elected to defer a total of $72,000 (4% of payroll). What is the maximum deductible contribution to the profit sharing plan BMF can make?

A)
$450,000
B)
$522,000
C)
$265,000
D)
$378,000

A

a

The maximum deductible contribution BMF can make is 25% of aggregate covered payroll (25% × $1,800,000 = $450,000).

LO 1.3.3

85
Q

Which of these is a type of defined contribution profit-sharing plan?

A)
Cash balance pension plan
B)
Money purchase pension plan
C)
Target benefit pension plan
D)
Employee stock ownership plan (ESOP)

A

D

A cash balance pension plan is a type of defined benefit pension plan. An ESOP is a defined contribution profit-sharing plan.

LO 1.2.1

85
Q

Which of the following are agencies that administer and ensure compliance with the federal laws that apply to qualified retirement plans?

Department of Labor (DOL)
Employee Retirement Income Security Agency (ERISA)
Internal Revenue Service (IRS)
Pension Benefit Guaranty Corporation (PBGC)
A)
I, II, and III
B)
I and III
C)
II and IV
D)
I, III, and IV

A

d

Under the reporting and disclosure requirements of the Employee Retirement Income Security Act of 1974 (ERISA), annual reports and summary plan descriptions are filed with the IRS (which provides copies to the DOL). An annual premium payment form is filed with the PBGC.

LO 1.1.1

86
Q

Company A has a participant-directed retirement plan that offers six investment options ranging from conservative to risky. Sam, an employee, had his account invested in the riskiest option. When he suffered severe losses in the account, he sued the plan fiduciary. Which of the following statements regarding the fiduciary’s liability to Sam is CORRECT?

A)
The fiduciary is responsible for only 20% of the loss suffered by Sam.
B)
The fiduciary is not responsible for the loss suffered by Sam because the investment options were sufficiently diverse and Sam chose the risky portfolio.
C)
The fiduciary is fully responsible for the loss to Sam because investing accounts in risky portfolios is not an appropriate choice for participants.
D)
The fiduciary is not responsible for the loss suffered by Sam but must replace the risky portfolio with a more conservative portfolio.

A

B

Investment options that cover the entire investment spectrum are appropriate. The participant chose the option and will benefit or suffer accordingly. However, the fiduciary must select quality investment choices.

LO 1.4.1

Think:
WJA has Aggressive all the way to Fixed Income models for clients to choose from. They make sure that the portfolios are adequately diversified, and it’s up to the clients how exposed they are willing to be to risk.

86
Q

Which of the following is CORRECT about the permitted disparity (Social Security integration) rules for qualified plans?

An integration level for a defined contribution plan that exceeds the current year’s taxable wage base may be selected.
The permitted disparity level in a defined benefit plan must be reduced for early retirement.
It is no longer possible to have a defined benefit formula in which lower-paid employees receive no benefit.
The permitted disparity under a defined benefit plan, using covered compensation as the integration point, is the Social Security wage base averaged over a maximum of 35 service years.
A)
II and IV
B)
II, III, and IV
C)
I and III
D)
I, III, and IV

A

b

Statements II, III, and IV correctly state the following integration rules. The permitted disparity level must be reduced if a participant retires early. It is no longer possible to have an integrated excess plan that does not provide any benefits (or contributions) to employees with wages below the integration level. This was permitted under prior law. The excess benefit percentage is calculated on a per-year-of-service basis (over a maximum of 35 years). Statement I is incorrect because the integration level selected cannot exceed the current year’s taxable wage base; however, it may be less.

86
Q

Gene wants to establish a qualified plan for his business to provide employees of the company with the ability to save for retirement. Which of the following plans is a qualified plan?

Profit-sharing plan
Simplified employee pension (SEP) plan
SIMPLE IRA
SARSEP plan
A)
II and III
B)
IV only
C)
I only
D)
I, II, III, and IV

A

I only

Of the plans listed, only the profit-sharing plan is a qualified plan. The SIMPLE IRA and the SEP plan are tax-advantaged plans, and the Section 457 plan is a nonqualified plan.

87
Q

Pension Benefit Guaranty Corporation (PBGC) insurance coverage is required for which of these types of retirement plans?

A)
Target benefit plans
B)
Money purchase plans
C)
Profit-sharing plans
D)
Traditional defined benefit plans

A

The answer is traditional defined benefit plans. Money purchase pension plans, profit-sharing plans, and target benefit pension plans do not require PBGC insurance because they are forms of defined contribution plans. Only defined benefit pension plans (traditional defined benefit plans and cash balance plans) require the payment of PBGC insurance premiums.

LO 1.1.1

87
Q

The plan administrator

A)
is not responsible for the Summary Plan Description (SPD).
B)
of a small company (50 employees or less) must file a detailed annual report (IRS Form 5500) each year.
C)
must do all of these.
D)
must comply with the reporting and disclosure requirements under the Employee Retirement Income Security Act (ERISA).

A

D

A trustee or insurance company may take over a significant number of the plan administrator’s duties; however, the plan administrator is still responsible for plan compliance with IRS and DOL regulations. Small plans may not be required to file Form 5500 each year.

LO 1.4.1

87
Q

Which of the following transactions between a qualified plan and a disqualified person are prohibited transactions, as defined by ERISA?

The employer’s purchase of a mortgage note in default from the plan for more than fair market value
The sale of undeveloped land to a qualified plan for a bargain price
The acquisition of employer stock by a profit-sharing plan for full and adequate consideration
A loan to a 100% shareholder-participant in an amount proportionate to the amounts available to other plan participants, as specified in the plan documents
A)
I, II, and III
B)
I and II
C)
II, III, and IV
D)
II and III

A

b

Options I and II, respectively, are prohibited transactions for the following reasons. The sale or exchange of property (i.e., a mortgage note in this situation) between a plan and a disqualified person results in a prohibited transaction. The sale of the undeveloped land is a prohibited transaction for the same reason. The federal courts have held that an in-kind contribution of employer-owned property to a plan in satisfaction of the employer’s funding obligation is a prohibited transaction (i.e., a sale or exchange).

Options III and IV are not prohibited transactions. ERISA specifically exempts the acquisition or sale of qualifying employer securities by an individual account plan (such as a profit-sharing plan) from prohibited transaction treatment. Loans in accordance with plan provisions are available to shareholder-participants who are disqualified persons.

LO 1.4.1

88
Q

An employer establishing a qualified plan may apply to the IRS for a favorable opinion that the plan provisions meet the requirements of the Tax Code. This type of opinion is known as

A)
a Summary Annual Report (SAR).
B)
an advance determination letter.
C)
a Summary of Material Modification (SMM).
D)
a Summary Plan Description (SPD).

A

An advance determination letter is an opinion from the IRS that the plan provisions satisfy the requirements of a qualified plan.

LO 1.4.1

89
Q

In a defined contribution plan, if the integration level is the Social Security taxable wage base for retirement, and the base contribution percentage is 7%, the excess contribution percentage can be as high as

A)
10.0%.
B)
13.4%.
C)
12.7%.
D)
15.3%.

A

C

The maximum permitted disparity, or the difference between the base contribution percentage and the excess contribution percentage, cannot exceed the lesser of the base contribution percentage or 5.7% (the Social Security tax rate attributable to OASDI). Therefore, 7% + 5.7% equals 12.7%.

Basically it’s 2*7% vs 7%+5.7% (LESSER OF)

LO 1.3.3

89
Q

In a defined contribution plan, if the integration level is less than the Social Security taxable wage base, what occurs?

Larger contributions will be made on behalf of employees whose compensation is above the lower integration level.
The plan’s cost to the employer increases as the integration level is reduced.
The permitted disparity between the maximum excess contribution percentage and the base contribution percentage is reduced.
Employer contributions must be adjusted to an integration level of $160,200 (2023) to achieve optimum integration.
A)
II and IV
B)
I and III
C)
I, II, and III
D)
III, and IV

A

Only Statement IV is incorrect. An integration level below the Social Security taxable wage base allows larger contributions for higher-paid workers and costs the employer more money. Optimum integration is generally just above the compensation level of the highest-paid nonowner employee.

LO 1.3.3